Convergenza doppia

Cannelloni1
Buonasera, sto lavorando a questo problema da diversi giorni ormai ma non trovo soluzione.
Sia $X$ spazio vettoriale e $||\cdot||_1$, $||\cdot||_2$ due norme qualsiasi che rendono $X$ completo (cioè, sia $(X,||\cdot||_1)$ che $(X,||\cdot||_2)$ sono spazi metrici completi). E' possibile trovare una successione $x_n\in X$ tale che
\[
x_n\overset{||\cdot||_1}{\longrightarrow} 0
\]
mentre
\[
x_n\overset{||\cdot||_2}{\longrightarrow} y\neq 0?
\]

Onestamente io spero di no.
So che c'è un teorema che dice che tutte le norme che rendono $X$ Banach sono equivalenti, ma non posso usarlo (perché lo sto dimostrando con questo fatto...).

L'unica cosa che ho dimostrato è che se esiste una tale $x_n$ allora deve esistere anche una $z_n$ tale che $||z_n||_1\rightarrow 0$ e $||z_n||_2\rightarrow\infty$

Risposte
Grazie della bella domanda! La risposta è sì. Se le due norme sono equivalenti allora no. Ma non c'è nessun motivo per cui le due norme siano equivalenti.
La risposta di Bill Johnson a questa domanda è un esempio di quello che cerchi:
https://mathoverflow.net/questions/53971/example-of-sequences-with-different-limits-for-two-norms

Ti espando un pochino qua la sua risposta per chiarezza. Inoltre sebbene nella domanda di MO non sia richiesto che le due norme rendano \(X\) completo, l'esempio di Bill Johnson va benissimo per la tua domanda. Infatti le due norme che considera rendono \(X\) completo per entrambe.

Prendi \( X=\ell^2 \). Sia \( \left\| \xi \right\|_2 = \sum_{j \in \mathbb{N}} \left|\xi_j \right|^2 \). Abbiamo che \( (\ell^2, \left\| \cdot \right\|_2) \) è completo. Ti ricordo che una base di Schauder estende il concetto di base di Hamel, se in uno spazio vettoriale una base di Hamel ti permette di esprimere ogni vettore in modo unico come combinazione lineare finita di elementi della tua base, in una base di Schauder, ogni vettore lo esprimi come una combinazione lineare anche infinita.
Prendi ora \( (e_n)_{n \geq 1} \) la base di Schauder di vettori ortonormali standard di \( \ell^2\). Definisci \( x_n :=e_1 + \frac{e_n}{n} \).
Definisci l'operatore \(T\) dalla seguente regola \( x_n \mapsto \frac{e_n}{n} \), per ogni \(n \geq 1\). Ora estendi \(T \) ad un isomorfismo lineare \( T : \ell_2 \to \ell_2 \). Per fare questo è sufficiente estendere gli insiemi \( \{x_n : n \geq 1 \}\), \( \{ \frac{e_n}{n} : n \geq 1 \} \) a delle basi di Hamel. Grazie al assioma della scelta (AC) puoi prendere \( \mathcal{B}_1, \mathcal{B}_2 \subseteq \ell^2 \) tale che \( \mathcal{B}_1 \cup \{ x_n : n \geq 1 \} \) e \( \mathcal{B}_2 \cup \{ \frac{e_n}{n} : n \geq 1 \} \) sono delle basi di Hamel. (Stando al seguente link, difficilmente le trovi esplicitamente le basi di Hamel https://math.stackexchange.com/questions/194189/a-hamel-basis-for-ellp). Ora estendi \(T\) in una biiezione tra queste due basi, e poi lo estendi per linearità a tutto \( \ell^2\).

Definiamo ora \( \left\| \cdot \right\|_1 \) da \( \left\| \xi \right\|_1 := \left\| T \xi \right\|_2 \), per ogni \( \xi \in \ell^2 \). E' facile dimostrare che \( (\ell^2, \left\| \right\|_1) \) è completo.
Prendi \( (\xi_n)_{n \geq 1} \) essere una successione di Cauchy in \( (\ell^2, \left\| \cdot \right\|_1) \). Allora \( (T\xi_n)_{n \geq 1} \) è di Cauchy in \( (\ell^2, \left\| \cdot \right\|_2) \). Per completezza di \( (\ell^2, \left\| \cdot \right\|_2) \) abbiamo che \(T\xi_n \to \eta \), per qualche \(\eta \in \ell^2 \). Per costruzione \(T\) è un isomorfismo, dunque \(\eta=T\xi\) per qualche \( \xi \in \ell^2\). Pertanto deduciamo che
\[ \left\| \xi_n - \xi \right\|_1 = \left\| T(\xi_n-\xi) \right\|_2 = \left\| T \xi_n - \eta \right\|_2 \to 0 \]
e dunque \( (\xi_n)_{n \geq 1} \) converge e questo significa che \( (\ell^2 , \left\| \right\|_1)\) è completo.
Sia come prima \(x_n := e_1 + \frac{e_n}{n} \). Abbiamo che
\[ \left\|x_n - e_1 \right\|_2 = \left\| \frac{e_n}{n} \right\|_2 \to 0, \]
mentre
\[ \left\|x_n - 0 \right\|_1 = \left\| T(x_n - 0) \right\|_2 = \left\| \frac{e_n}{n} \right\|_2 \to 0 \]
dunque
\[ x_n \overset{\left\| \cdot \right\|_2}{\longrightarrow} e_1 \]
e
\[ x_n \overset{\left\| \cdot \right\|_1}{\longrightarrow} 0 \]

"Cannelloni":

Onestamente io spero di no.
So che c'è un teorema che dice che tutte le norme che rendono $X$ Banach sono equivalenti, ma non posso usarlo (perché lo sto dimostrando con questo fatto...).

Faccio una piccola precisazione, perché mi ha confuso la mia risposta :lol:
In effetti mi stavo domandando come fosse possibile che uno spazio di Banach possedesse due norme non equivalenti. Ma come?! Le norme sugli spazi di Banach non sono equivalenti? No!
Infatti stai attento che quello che hai detto è falso! In effetti l'esempio di cui sopra è un esempio di spazio di Banach con due norme non equivalenti. Il teorema che dici tu è una consequenza del teorema della mappa aperta. E dice la seguente cosa

Siano \( \left\| \cdot \right\|_1 \) e \( \left\| \cdot \right\|_2 \) due norme su uno spazio vettoriale \(X \neq \{ 0\} \), tale che \( ( X, \left\| \cdot \right\|_1) \) e \( (X, \left\| \cdot \right\|_2 ) \) siano spazi di Banach.
Se esiste \(C >0 \) tale che
\[ \forall x \in X, \left\| x \right\|_2 \leq C \left\| x \right\|_1 \]
allora esiste \(D > 0 \) tale che
\[ \forall x \in X, \left\| x \right\|_1 \leq D \left\| x \right\|_2. \]
E le due norme sono equivalenti.

Però l'ipotesi che esiste \( C > 0 \) tale che \( \forall x \in X, \left\| x \right\|_2 \leq C \left\| x \right\|_1\) potrebbe non essere soddisfatta, in effetti se non viene soddisfatta si dice che le norme sono incomparabili. E quindi in uno spazio di Banach tutte le norme sono o incomparabili oppure equivalenti. Le due norme del esempio precedenti sono incomparabili. Due norme invece sono equivalenti se esistono \(A,B > 0 \) tale che \( \forall x \in X \) abbiamo che \( A \left\| x \right\|_1 \leq \left\| x \right\|_2 \leq B \left\| x \right\|_1 \). Ora se \(X\) è uno spazio vettoriale e \( (X, \left\| \cdot \right\|_2) \) e \( (X, \left\| \cdot \right\|_1) \) sono di Banach allora le seguenti due cose sono equivalenti:
1) Se \( \left\| x_n -x \right\|_2 \to 0 \) e \( \left\| x_n -y\right\|_1 \to 0 \) implica che \( y=x \).
2) \( \left\| \cdot \right\|_2 \) e \( \left\| \cdot \right\|_1 \) sono norme equivalenti.

\( 1 \Rightarrow 2\) essenzialmente perché l'identità (essendo continua) è un operatore lineare limitato. Mentre \( 2 \Rightarrow 1 \) essenzialmente per il teorema dei Carabinieri, in effetti se sono equivalenti allora \[ A \left\| x_n -x\right\|_1 \leq \left\| x_n -x\right\|_2 \leq B \left\| x_n -x\right\|_1 \]
Da \( \left\| x_n -x\right\|_1 \to 0 \) deduciamo che \( \left\| x_n -x\right\|_2 \to 0 \). Ora poiché gli spazi di Banach, essendo spazi metrici, sono di Hausdorff e quindi deduciamo che \( y=x \).

Ma cosa esattamente non funziona di questo argomento nel esempio precedente? Di sicuro la mappa identità è discontinua. Ti ricordo che \( L : X \to Y \) è continuo in \(x\) come operatore tra due spazi normati se e solo se \( L x_n \to L x \) ogni volta che \(x_n \to x \). Credo anche che \(T\) non è limitato, altrimenti le norme dovrebbero essere comparabili e quindi l'operatore \(T\) è un isomorfismo lineare non continuo. Nota infatti

Sia \(T: (\ell^2, \left\| \cdot \right\|_2 ) \to (\ell^2,\left\| \cdot \right\|_1 ) \), \( \xi \mapsto T \xi \) come nel esempio del mio messaggio precedente. Prendi ora \( \xi_n \to \xi \) rispetto a \( \left\| \cdot \right\|_2 \), i.e. \( \left\| \xi_n - \xi \right\|_2 \to 0 \). Come fai a concludere che \( \left\| T \xi_n - T \xi \right\|_1 \to 0 \) ?? Non credo che tu possa, perché \(T\) è discontinuo! Se avessimo invece che \( \left\| \xi_n - \xi \right\|_1 \to 0 \) allora sì \( \left\| T \xi_n - T \xi \right\|_2 \to 0 \).

Qua c'è un altro modo:
https://mathoverflow.net/questions/184464/are-banach-space-norms-up-to-equivalence-unique

dissonance
Non so se ho capito bene. Se questo fatto è vero:

So che c'è un teorema che dice che tutte le norme che rendono $X$ Banach sono equivalenti,

allora chiaramente questo non può mai succedere:
"Cannelloni":
Sia $X$ spazio vettoriale e $||\cdot||_1$, $||\cdot||_2$ due norme qualsiasi che rendono $X$ completo [...]
\[
x_n\overset{||\cdot||_1}{\longrightarrow} 0
\]
mentre
\[
x_n\overset{||\cdot||_2}{\longrightarrow} y\neq 0?
\]

Infatti, se due norme sono equivalenti, allora hanno le stesse successioni convergenti, come è ovvio.

Ma mi sembra di capire, dagli interventi di 3m0o, che il fatto di cui sopra NON sia vero. Il che mi pare plausibile.

"dissonance":
Non so se ho capito bene. Se questo fatto è vero:

So che c'è un teorema che dice che tutte le norme che rendono $X$ Banach sono equivalenti,



Ma mi sembra di capire, dagli interventi di 3m0o, che il fatto di cui sopra NON sia vero. Il che mi pare plausibile.

Esatto, non mi sembra vero neanche a me! Devono essere norme comparabili pure. Cosa che non è immediato.
Ma non ho capito, ti sembra plausibile che sia vero o sia falso il claim delle norme equivalenti?

dissonance
Eh eh, a rileggermi mi accorgo che non era chiaro. Mi pare plausibile quello che dici tu 3m0o. Ovvero, penso che deve esistere uno spazio vettoriale che è di Banach rispetto a due norme non equivalenti.

P.S.: il tuo esempio dovrebbe essere corretto. Solo che non riesco a capire come fai a concludere che \(\lVert\cdot\rVert_1\) sia una norma di Banach.

"3m0o":

Definiamo ora \( \left\| \cdot \right\|_1 \) da \( \left\| \xi \right\|_1 := \left\| T \xi \right\|_2 \), per ogni \( \xi \in \ell^2 \). E' facile dimostrare che \( (\ell^2, \left\| \right\|_1) \) è completo.
Prendi \( (\xi_n)_{n \geq 1} \) essere una successione di Cauchy in \( (\ell^2, \left\| \cdot \right\|_1) \). Allora \( (T\xi_n)_{n \geq 1} \) è di Cauchy in \( (\ell^2, \left\| \cdot \right\|_2) \). Per completezza di \( (\ell^2, \left\| \cdot \right\|_2) \) abbiamo che \(T\xi_n \to \eta \), per qualche \(\eta \in \ell^2 \). Per costruzione \(T\) è un isomorfismo, dunque \(\eta=T\xi\) per qualche \( \xi \in \ell^2\). Pertanto deduciamo che
\[ \left\| \xi_n - \xi \right\|_1 = \left\| T(\xi_n-\xi) \right\|_2 = \left\| T \xi_n - \eta \right\|_2 \to 0 \]
e dunque \( (\xi_n)_{n \geq 1} \) converge e questo significa che \( (\ell^2 , \left\| \right\|_1)\) è completo.


Sia \( (\ell^2, \left\| \cdot \right\|_1) \) questo spazio vettoriale normato, e \(d_1(x,y)= \left\| x-y \right\|_1 \) la metrica indotta allora questo dovrebbe dimostrare che \( (\ell^2,d_1) \) è completo, e per definizione \( (\ell^2, \left\| \cdot \right\|_1) \) è uno spazio di Banach.
Se c'è qualche passaggio che non capisci posso provare a chiarire meglio.

Cannelloni1
Grazie a tutti delle risposte; mi torna tutto.
Mi dispiace per la confusione creata, sono d'accordo: non tutte le norme sono equivalenti, ma se vale $||\cdot||_1\leq C||\cdot||_2$ allora sono equivalenti per teorema della mappa aperta. Il fatto è che a lezione il professore diede questo esercizio senza l'ipotesi aggiuntiva e addirittura costruì il controesempio dicendo però che questo controesempio era fasullo e lasciò come esercizio capire come mai. A quanto pare, semplicemente, si deve essere confuso. Capita a tutti :)

"Cannelloni":
Grazie a tutti delle risposte; mi torna tutto.
Mi dispiace per la confusione creata, sono d'accordo: non tutte le norme sono equivalenti, ma se vale $||\cdot||_1\leq C||\cdot||_2$ allora sono equivalenti per teorema della mappa aperta. Il fatto è che a lezione il professore diede questo esercizio senza l'ipotesi aggiuntiva e addirittura costruì il controesempio dicendo però che questo controesempio era fasullo e lasciò come esercizio capire come mai. A quanto pare, semplicemente, si deve essere confuso. Capita a tutti :)

Mah, se il tuo prof ha fatto questo esempio e ha detto che non funziona, magari è così. Chiederei a lui per conferma, e poi fai sapere che sono curioso.

Rispondi
Per rispondere a questa discussione devi prima effettuare il login.